Integrating a Definite Integral with Trigonometric Functions

  • Thread starter Thread starter utkarsh009
  • Start date Start date
  • Tags Tags
    Integration
Click For Summary
The discussion centers on evaluating the definite integral ∫dt/(t^2 + 2t cos a + 1) from 0 to 1, with the constraint 0 < a < π. The initial approach involved substituting t = sin a and changing the limits of integration, but confusion arose regarding the manipulation of the cosec a term. A suggestion was made to use a different substitution, t = cos(a), to simplify the integral. Ultimately, the original poster found a solution but sought further assistance on a related problem. The conversation highlights common challenges in integrating trigonometric functions and the importance of proper substitution techniques.
utkarsh009
Messages
46
Reaction score
0

Homework Statement


∫dt/(t^2 +2tcos a + 1)
(Limits of the integral are from 0 to 1)
(0<a<π)

Homework Equations

The Attempt at a Solution


Put t=sin a
dt=cosa da
∫dt/(t^2 +2tcos a + 1) = ∫cos a da/(sin^2 a + sin 2a + 1) [ limits of integration changed to 0 to π/2]
= ((cosec a)/2) ∫sin 2a da/(sin^2 a + sin 2a + 1)

I couldn't figure out what to do next... Please help!
 
Last edited by a moderator:
Physics news on Phys.org
utkarsh009 said:

Homework Statement


∫dt/(t^2 +2tcos a + 1)
(Limits of the integral are from 0 to 1)
(0<a<π)

Homework Equations

The Attempt at a Solution


Put t=sin a
dt=cosa da
∫dt/(t^2 +2tcos a + 1) = ∫cos a da/(sin^2 a + sin 2a + 1) [ limits of integration changed to 0 to π/2]
= ((cosec a)/2) ∫sin 2a da/(sin^2 a + sin 2a + 1)
I don't know what you did in this last step, but you can't bring csc(a) out as if it were a constant.
utkarsh009 said:
I couldn't figure out what to do next... Please help!
 
Oh yes... By mistake i typed the integration symbol after it. Cosec a should be inside the integral. So, how should i proceed??
 
I would try a different substitution -- let t = cos(a).That should get you an integral that's easier to do.
 
Oh yah... Thanks ... I solved it... But have a look at this file... I couldn't get an answer for this...
New Doc 1_1.jpg
 
Question: A clock's minute hand has length 4 and its hour hand has length 3. What is the distance between the tips at the moment when it is increasing most rapidly?(Putnam Exam Question) Answer: Making assumption that both the hands moves at constant angular velocities, the answer is ## \sqrt{7} .## But don't you think this assumption is somewhat doubtful and wrong?

Similar threads

  • · Replies 5 ·
Replies
5
Views
2K
  • · Replies 3 ·
Replies
3
Views
2K
  • · Replies 6 ·
Replies
6
Views
1K
  • · Replies 9 ·
Replies
9
Views
2K
  • · Replies 9 ·
Replies
9
Views
1K
  • · Replies 1 ·
Replies
1
Views
2K
  • · Replies 8 ·
Replies
8
Views
2K
  • · Replies 6 ·
Replies
6
Views
1K
  • · Replies 3 ·
Replies
3
Views
2K
  • · Replies 9 ·
Replies
9
Views
1K